Difference between revisions of "2023 AMC 12B Problems/Problem 22"

m
 
Line 14: Line 14:
 
Substituting <math>a= 0</math> we find <cmath>2f(0) = 2f(0)^2 \implies f(0) \in \{0, 1\}.</cmath>
 
Substituting <math>a= 0</math> we find <cmath>2f(0) = 2f(0)^2 \implies f(0) \in \{0, 1\}.</cmath>
 
This gives <cmath>f(2a) = 2f(a)^2 - f(0) \geq 0-1</cmath>
 
This gives <cmath>f(2a) = 2f(a)^2 - f(0) \geq 0-1</cmath>
Plugging in <math>a = \frac{1}{2}</math> implies <math>f(1) \geq -1</math>, so answer choice <math>\boxed{\textbf{(E) }}</math> is impossible.  
+
Plugging in <math>a = \frac{1}{2}</math> implies <math>f(1) \geq -1</math>, so answer choice <math>\boxed{\textbf{(E) -2}}</math> is impossible.  
  
 
~AtharvNaphade
 
~AtharvNaphade
Line 71: Line 71:
 
The range of <math>\cosh(ay)</math> is <math>[1,\infty)</math> so choice <math>D</math> is incorrect.
 
The range of <math>\cosh(ay)</math> is <math>[1,\infty)</math> so choice <math>D</math> is incorrect.
  
Therefore, the remaining answer is choice <math>\boxed{(E)}.</math>
+
Therefore, the remaining answer is choice <math>\boxed{\textbf{(E) -2}}.</math>
  
 
~kxiang
 
~kxiang
Line 78: Line 78:
 
https://youtu.be/UML6WmL0mNk
 
https://youtu.be/UML6WmL0mNk
  
==Video Solution==
+
==Video Solution 2==
  
 
https://youtu.be/GK0ZpjxbwLw
 
https://youtu.be/GK0ZpjxbwLw

Latest revision as of 09:50, 4 December 2023

Problem

A real-valued function $f$ has the property that for all real numbers $a$ and $b,$ \[f(a + b)  + f(a - b) = 2f(a) f(b).\] Which one of the following cannot be the value of $f(1)?$

$\textbf{(A) } 0 \qquad \textbf{(B) } 1 \qquad \textbf{(C) } -1 \qquad \textbf{(D) } 2 \qquad \textbf{(E) } -2$

Solution 1

Substituting $a = b$ we get \[f(2a) + f(0) = 2f(a)^2\] Substituting $a= 0$ we find \[2f(0) = 2f(0)^2 \implies f(0) \in \{0, 1\}.\] This gives \[f(2a) = 2f(a)^2 - f(0) \geq 0-1\] Plugging in $a = \frac{1}{2}$ implies $f(1) \geq -1$, so answer choice $\boxed{\textbf{(E) -2}}$ is impossible.

~AtharvNaphade

Solution 2

First, we set $a \leftarrow 0$ and $b \leftarrow 0$. Thus, the equation given in the problem becomes $[ f(0) + f(0) = 2 f(0) \times f(0) . ]$

Thus, $f(0) = 0$ or 1.

Case 1: $f(0) = 0$.

We set $b \leftarrow 0$. Thus, the equation given in the problem becomes $[ 2 f(a) = 0 . ]$

Thus, $f(a) = 0$ for all $a$.

Case 2: $f(0) = 1$.

We set $b \leftarrow a$. Thus, the equation given in the problem becomes \[[ f(2a) + 1 = 2 \left( f(a) \right)^2. ]\]

Thus, for any $a$, \begin{align*} f(2a) & = -1 + 2 \left( f(a) \right)^2 \\ & \geq -1 . \end{align*}

Therefore, an infeasible value of $f(1)$ is $\boxed{\textbf{(E) -2}}.$

~Steven Chen (Professor Chen Education Palace, www.professorchenedu.com)

Solution 3

The relationship looks suspiciously like a product-to-sum identity. In fact, \[\cos(\alpha)\cos(\beta) = \frac{1}{2}(\cos(\alpha-\beta)+\cos(\alpha+\beta))\] which is basically the relation. So we know that $f(x) = \cos(x)$ is a valid solution to the function. However, if we define $x=ay,$ where $a$ is arbitrary, the above relation should still hold for $f(x) = \cos(ay) = \cos(a(1))$ so any value in $[-1,1]$ can be reached, so choices $A,B,$ and $C$ are incorrect.

In addition, using the similar formula for hyperbolic cosine, we know \[\cosh(\alpha)\cosh(\beta) = \frac{1}{2}(\cosh(\alpha-\beta)+\cosh(\alpha+\beta))\] The range of $\cosh(ay)$ is $[1,\infty)$ so choice $D$ is incorrect.

Therefore, the remaining answer is choice $\boxed{\textbf{(E) -2}}.$

~kxiang

Video Solution 1 by OmegaLearn

https://youtu.be/UML6WmL0mNk

Video Solution 2

https://youtu.be/GK0ZpjxbwLw

~Steven Chen (Professor Chen Education Palace, www.professorchenedu.com)


See also

2023 AMC 12B (ProblemsAnswer KeyResources)
Preceded by
Problem 21
Followed by
Problem 23
1 2 3 4 5 6 7 8 9 10 11 12 13 14 15 16 17 18 19 20 21 22 23 24 25
All AMC 12 Problems and Solutions

The problems on this page are copyrighted by the Mathematical Association of America's American Mathematics Competitions. AMC logo.png